Solve Max Volume of Trapezoidal Prism | Optimization Problem

  • Thread starter Thread starter jen333
  • Start date Start date
Click For Summary
The discussion revolves around optimizing the volume of a trapezoidal prism by determining the angle theta that maximizes volume. The prism has a length of 20m, with specific dimensions for the bases and height defined in terms of x. The user seeks assistance in solving the derivative equation for x, which leads to a complex expression involving square roots. Suggestions include using trial and error or graphing to find solutions, as well as referencing Ferrari's method for solving quartic equations. The conversation highlights the challenges in deriving a solution and the need for effective mathematical strategies.
jen333
Messages
58
Reaction score
0
Hi! I have a tricky optimization question here. Not so much setting it up, but solving the derivative for x. (if i did set it up right)

Question: A what angle of theta is the trough (trapezoidal prism) to allow maximum volume?
Unfortunately, i don't have any program that allows me to draw out the diagram (not even paint) so hopefully my symbols/letters diagram will substitute

Face of prism, prism is 20m in length
_______
\ |____| /

the shorter base (b1) is 1m, while the hypotenuse of the end triangles are 1m as well
height is assigned as x while the shorter side of the triangle is (1-x^2)^(1/2)
(the longer base) b2=1+2(1-x^2)^(1/2 theta is the angle between x and the hypotenuse which i know can be solved by cos(theta)=x

so: V=lh (b1+b2)
2
therefore: V=20x(2+2(1-x^2)^(1/2))/2

V'=-20x^2+20(1-x^2)^(1/2)+20-20x^2
0= -40x^2+20(1-x^2)^(1-2)+20

IF i did everything correctly, my question is, how would I solve for x with the root (1-x^2) in the way? I tried squaring all terms, bu that only left me with X^4 and X^2.

help appreciated! thanks.
 
Last edited:
Physics news on Phys.org
Question: A clock's minute hand has length 4 and its hour hand has length 3. What is the distance between the tips at the moment when it is increasing most rapidly?(Putnam Exam Question) Answer: Making assumption that both the hands moves at constant angular velocities, the answer is ## \sqrt{7} .## But don't you think this assumption is somewhat doubtful and wrong?

Similar threads

Replies
3
Views
2K
  • · Replies 34 ·
2
Replies
34
Views
3K
  • · Replies 9 ·
Replies
9
Views
2K
  • · Replies 9 ·
Replies
9
Views
2K
  • · Replies 2 ·
Replies
2
Views
1K
Replies
2
Views
2K
  • · Replies 6 ·
Replies
6
Views
3K
  • · Replies 4 ·
Replies
4
Views
2K
  • · Replies 1 ·
Replies
1
Views
2K
Replies
4
Views
2K